Reducing system of 10 inequalities











up vote
1
down vote

favorite












I have the following system of inequalities of real variables



sys = 0.383706 x1 + 0.720204 x2 + 1.4568 x3 - 0.000244875 y >   0 && -0.0941312 x1 - 0.176681 x2 - 0.357383 x3 + 0.0000592689 y >   0 && 1.41819 x1 + 3.079 x2 + 2.53801 x3 - 0.00182772 y >   0 && -0.00258971 x1 - 0.00562247 x2 - 0.00463458 x3 +    3.93843*10^-6 y > 0 &&  0.129511 x1 + 0.214698 x2 + 0.286739 x3 - 0.0000795243 y >   0 && -0.660462 x1 - 1.09489 x2 - 1.46227 x3 + 0.000405426 y > 0 &&  0.351966 x1 + 3.44019 x2 + 7.59863 x3 + 0.00257072 y >   0 && -0.00265509 x1 - 0.0259514 x2 - 0.0573209 x3 -    0.0000199557 y > 0 &&  1.34471 x1 + 2.58639 x2 + 3.39561 x3 + 0.0000742173 y >   0 && -0.0314846 x1 - 0.0605571 x2 - 0.079504 x3 - 1.69328*10^-6 y >   0


I want to Reduce the system in order to see if it admits solutions or not. However, the command Reduce takes too much time and gets stuck. How can you check if this system admits solutions?



I have tried with FindInstance[sys,{x1,x2,x3,y},Reals] which returns {}. However, I am not sure this is a fully proof that the system does not admit solution. Indeed, If I run the command on a subsystem of inequalities I experience the following behaviour



FindInstance[sys[[1 ;; 4]], {x1, x2, x3, y}, Reals]
(* {} *)


but If I ask for more instances, Mathematica found the points



FindInstance[sys[[1 ;; 4]], {x1, x2, x3, y}, Reals, 2]
(*{{x1 -> -33., x2 -> 13.5619, x3 -> 1.98727, y -> 0.255564}, {x1 -> -51., x2 -> 20.9595, x3 -> 3.07117, y -> 0.601467}}*)


So, the subsystem [[1;;4]] admits at least a solution. I have run FindInstance[sys, {x1, x2, x3, y}, Reals, 2] for all the night and got the output {}. Still, is this a proof that the system does not admits solution?










share|improve this question


















  • 1




    Reduce[Rationalize[sys, 2^-12], {x1, x2, x3, y}, Reals] gives False.
    – kglr
    yesterday






  • 1




    and RegionIntersection @@ (ImplicitRegion[#, {x1, x2, x3, y}] & /@ (List @@ sys)) gives EmptyRegion[4].
    – kglr
    yesterday















up vote
1
down vote

favorite












I have the following system of inequalities of real variables



sys = 0.383706 x1 + 0.720204 x2 + 1.4568 x3 - 0.000244875 y >   0 && -0.0941312 x1 - 0.176681 x2 - 0.357383 x3 + 0.0000592689 y >   0 && 1.41819 x1 + 3.079 x2 + 2.53801 x3 - 0.00182772 y >   0 && -0.00258971 x1 - 0.00562247 x2 - 0.00463458 x3 +    3.93843*10^-6 y > 0 &&  0.129511 x1 + 0.214698 x2 + 0.286739 x3 - 0.0000795243 y >   0 && -0.660462 x1 - 1.09489 x2 - 1.46227 x3 + 0.000405426 y > 0 &&  0.351966 x1 + 3.44019 x2 + 7.59863 x3 + 0.00257072 y >   0 && -0.00265509 x1 - 0.0259514 x2 - 0.0573209 x3 -    0.0000199557 y > 0 &&  1.34471 x1 + 2.58639 x2 + 3.39561 x3 + 0.0000742173 y >   0 && -0.0314846 x1 - 0.0605571 x2 - 0.079504 x3 - 1.69328*10^-6 y >   0


I want to Reduce the system in order to see if it admits solutions or not. However, the command Reduce takes too much time and gets stuck. How can you check if this system admits solutions?



I have tried with FindInstance[sys,{x1,x2,x3,y},Reals] which returns {}. However, I am not sure this is a fully proof that the system does not admit solution. Indeed, If I run the command on a subsystem of inequalities I experience the following behaviour



FindInstance[sys[[1 ;; 4]], {x1, x2, x3, y}, Reals]
(* {} *)


but If I ask for more instances, Mathematica found the points



FindInstance[sys[[1 ;; 4]], {x1, x2, x3, y}, Reals, 2]
(*{{x1 -> -33., x2 -> 13.5619, x3 -> 1.98727, y -> 0.255564}, {x1 -> -51., x2 -> 20.9595, x3 -> 3.07117, y -> 0.601467}}*)


So, the subsystem [[1;;4]] admits at least a solution. I have run FindInstance[sys, {x1, x2, x3, y}, Reals, 2] for all the night and got the output {}. Still, is this a proof that the system does not admits solution?










share|improve this question


















  • 1




    Reduce[Rationalize[sys, 2^-12], {x1, x2, x3, y}, Reals] gives False.
    – kglr
    yesterday






  • 1




    and RegionIntersection @@ (ImplicitRegion[#, {x1, x2, x3, y}] & /@ (List @@ sys)) gives EmptyRegion[4].
    – kglr
    yesterday













up vote
1
down vote

favorite









up vote
1
down vote

favorite











I have the following system of inequalities of real variables



sys = 0.383706 x1 + 0.720204 x2 + 1.4568 x3 - 0.000244875 y >   0 && -0.0941312 x1 - 0.176681 x2 - 0.357383 x3 + 0.0000592689 y >   0 && 1.41819 x1 + 3.079 x2 + 2.53801 x3 - 0.00182772 y >   0 && -0.00258971 x1 - 0.00562247 x2 - 0.00463458 x3 +    3.93843*10^-6 y > 0 &&  0.129511 x1 + 0.214698 x2 + 0.286739 x3 - 0.0000795243 y >   0 && -0.660462 x1 - 1.09489 x2 - 1.46227 x3 + 0.000405426 y > 0 &&  0.351966 x1 + 3.44019 x2 + 7.59863 x3 + 0.00257072 y >   0 && -0.00265509 x1 - 0.0259514 x2 - 0.0573209 x3 -    0.0000199557 y > 0 &&  1.34471 x1 + 2.58639 x2 + 3.39561 x3 + 0.0000742173 y >   0 && -0.0314846 x1 - 0.0605571 x2 - 0.079504 x3 - 1.69328*10^-6 y >   0


I want to Reduce the system in order to see if it admits solutions or not. However, the command Reduce takes too much time and gets stuck. How can you check if this system admits solutions?



I have tried with FindInstance[sys,{x1,x2,x3,y},Reals] which returns {}. However, I am not sure this is a fully proof that the system does not admit solution. Indeed, If I run the command on a subsystem of inequalities I experience the following behaviour



FindInstance[sys[[1 ;; 4]], {x1, x2, x3, y}, Reals]
(* {} *)


but If I ask for more instances, Mathematica found the points



FindInstance[sys[[1 ;; 4]], {x1, x2, x3, y}, Reals, 2]
(*{{x1 -> -33., x2 -> 13.5619, x3 -> 1.98727, y -> 0.255564}, {x1 -> -51., x2 -> 20.9595, x3 -> 3.07117, y -> 0.601467}}*)


So, the subsystem [[1;;4]] admits at least a solution. I have run FindInstance[sys, {x1, x2, x3, y}, Reals, 2] for all the night and got the output {}. Still, is this a proof that the system does not admits solution?










share|improve this question













I have the following system of inequalities of real variables



sys = 0.383706 x1 + 0.720204 x2 + 1.4568 x3 - 0.000244875 y >   0 && -0.0941312 x1 - 0.176681 x2 - 0.357383 x3 + 0.0000592689 y >   0 && 1.41819 x1 + 3.079 x2 + 2.53801 x3 - 0.00182772 y >   0 && -0.00258971 x1 - 0.00562247 x2 - 0.00463458 x3 +    3.93843*10^-6 y > 0 &&  0.129511 x1 + 0.214698 x2 + 0.286739 x3 - 0.0000795243 y >   0 && -0.660462 x1 - 1.09489 x2 - 1.46227 x3 + 0.000405426 y > 0 &&  0.351966 x1 + 3.44019 x2 + 7.59863 x3 + 0.00257072 y >   0 && -0.00265509 x1 - 0.0259514 x2 - 0.0573209 x3 -    0.0000199557 y > 0 &&  1.34471 x1 + 2.58639 x2 + 3.39561 x3 + 0.0000742173 y >   0 && -0.0314846 x1 - 0.0605571 x2 - 0.079504 x3 - 1.69328*10^-6 y >   0


I want to Reduce the system in order to see if it admits solutions or not. However, the command Reduce takes too much time and gets stuck. How can you check if this system admits solutions?



I have tried with FindInstance[sys,{x1,x2,x3,y},Reals] which returns {}. However, I am not sure this is a fully proof that the system does not admit solution. Indeed, If I run the command on a subsystem of inequalities I experience the following behaviour



FindInstance[sys[[1 ;; 4]], {x1, x2, x3, y}, Reals]
(* {} *)


but If I ask for more instances, Mathematica found the points



FindInstance[sys[[1 ;; 4]], {x1, x2, x3, y}, Reals, 2]
(*{{x1 -> -33., x2 -> 13.5619, x3 -> 1.98727, y -> 0.255564}, {x1 -> -51., x2 -> 20.9595, x3 -> 3.07117, y -> 0.601467}}*)


So, the subsystem [[1;;4]] admits at least a solution. I have run FindInstance[sys, {x1, x2, x3, y}, Reals, 2] for all the night and got the output {}. Still, is this a proof that the system does not admits solution?







equation-solving inequalities findinstance






share|improve this question













share|improve this question











share|improve this question




share|improve this question










asked yesterday









apt45

506211




506211








  • 1




    Reduce[Rationalize[sys, 2^-12], {x1, x2, x3, y}, Reals] gives False.
    – kglr
    yesterday






  • 1




    and RegionIntersection @@ (ImplicitRegion[#, {x1, x2, x3, y}] & /@ (List @@ sys)) gives EmptyRegion[4].
    – kglr
    yesterday














  • 1




    Reduce[Rationalize[sys, 2^-12], {x1, x2, x3, y}, Reals] gives False.
    – kglr
    yesterday






  • 1




    and RegionIntersection @@ (ImplicitRegion[#, {x1, x2, x3, y}] & /@ (List @@ sys)) gives EmptyRegion[4].
    – kglr
    yesterday








1




1




Reduce[Rationalize[sys, 2^-12], {x1, x2, x3, y}, Reals] gives False.
– kglr
yesterday




Reduce[Rationalize[sys, 2^-12], {x1, x2, x3, y}, Reals] gives False.
– kglr
yesterday




1




1




and RegionIntersection @@ (ImplicitRegion[#, {x1, x2, x3, y}] & /@ (List @@ sys)) gives EmptyRegion[4].
– kglr
yesterday




and RegionIntersection @@ (ImplicitRegion[#, {x1, x2, x3, y}] & /@ (List @@ sys)) gives EmptyRegion[4].
– kglr
yesterday










1 Answer
1






active

oldest

votes

















up vote
3
down vote



accepted










Reduce[Rationalize[sys, 2^-12], {x1, x2, x3, y}, Reals] // RepeatedTiming



{0.24878, False}




RegionIntersection @@ (ImplicitRegion[#, {x1, x2, x3, y}] & /@ (List @@ sys)) // 
RepeatedTiming



{0.0152, EmptyRegion[4]}







share|improve this answer





















    Your Answer





    StackExchange.ifUsing("editor", function () {
    return StackExchange.using("mathjaxEditing", function () {
    StackExchange.MarkdownEditor.creationCallbacks.add(function (editor, postfix) {
    StackExchange.mathjaxEditing.prepareWmdForMathJax(editor, postfix, [["$", "$"], ["\\(","\\)"]]);
    });
    });
    }, "mathjax-editing");

    StackExchange.ready(function() {
    var channelOptions = {
    tags: "".split(" "),
    id: "387"
    };
    initTagRenderer("".split(" "), "".split(" "), channelOptions);

    StackExchange.using("externalEditor", function() {
    // Have to fire editor after snippets, if snippets enabled
    if (StackExchange.settings.snippets.snippetsEnabled) {
    StackExchange.using("snippets", function() {
    createEditor();
    });
    }
    else {
    createEditor();
    }
    });

    function createEditor() {
    StackExchange.prepareEditor({
    heartbeatType: 'answer',
    convertImagesToLinks: false,
    noModals: true,
    showLowRepImageUploadWarning: true,
    reputationToPostImages: null,
    bindNavPrevention: true,
    postfix: "",
    imageUploader: {
    brandingHtml: "Powered by u003ca class="icon-imgur-white" href="https://imgur.com/"u003eu003c/au003e",
    contentPolicyHtml: "User contributions licensed under u003ca href="https://creativecommons.org/licenses/by-sa/3.0/"u003ecc by-sa 3.0 with attribution requiredu003c/au003e u003ca href="https://stackoverflow.com/legal/content-policy"u003e(content policy)u003c/au003e",
    allowUrls: true
    },
    onDemand: true,
    discardSelector: ".discard-answer"
    ,immediatelyShowMarkdownHelp:true
    });


    }
    });














     

    draft saved


    draft discarded


















    StackExchange.ready(
    function () {
    StackExchange.openid.initPostLogin('.new-post-login', 'https%3a%2f%2fmathematica.stackexchange.com%2fquestions%2f186279%2freducing-system-of-10-inequalities%23new-answer', 'question_page');
    }
    );

    Post as a guest















    Required, but never shown

























    1 Answer
    1






    active

    oldest

    votes








    1 Answer
    1






    active

    oldest

    votes









    active

    oldest

    votes






    active

    oldest

    votes








    up vote
    3
    down vote



    accepted










    Reduce[Rationalize[sys, 2^-12], {x1, x2, x3, y}, Reals] // RepeatedTiming



    {0.24878, False}




    RegionIntersection @@ (ImplicitRegion[#, {x1, x2, x3, y}] & /@ (List @@ sys)) // 
    RepeatedTiming



    {0.0152, EmptyRegion[4]}







    share|improve this answer

























      up vote
      3
      down vote



      accepted










      Reduce[Rationalize[sys, 2^-12], {x1, x2, x3, y}, Reals] // RepeatedTiming



      {0.24878, False}




      RegionIntersection @@ (ImplicitRegion[#, {x1, x2, x3, y}] & /@ (List @@ sys)) // 
      RepeatedTiming



      {0.0152, EmptyRegion[4]}







      share|improve this answer























        up vote
        3
        down vote



        accepted







        up vote
        3
        down vote



        accepted






        Reduce[Rationalize[sys, 2^-12], {x1, x2, x3, y}, Reals] // RepeatedTiming



        {0.24878, False}




        RegionIntersection @@ (ImplicitRegion[#, {x1, x2, x3, y}] & /@ (List @@ sys)) // 
        RepeatedTiming



        {0.0152, EmptyRegion[4]}







        share|improve this answer












        Reduce[Rationalize[sys, 2^-12], {x1, x2, x3, y}, Reals] // RepeatedTiming



        {0.24878, False}




        RegionIntersection @@ (ImplicitRegion[#, {x1, x2, x3, y}] & /@ (List @@ sys)) // 
        RepeatedTiming



        {0.0152, EmptyRegion[4]}








        share|improve this answer












        share|improve this answer



        share|improve this answer










        answered yesterday









        kglr

        171k8194399




        171k8194399






























             

            draft saved


            draft discarded



















































             


            draft saved


            draft discarded














            StackExchange.ready(
            function () {
            StackExchange.openid.initPostLogin('.new-post-login', 'https%3a%2f%2fmathematica.stackexchange.com%2fquestions%2f186279%2freducing-system-of-10-inequalities%23new-answer', 'question_page');
            }
            );

            Post as a guest















            Required, but never shown





















































            Required, but never shown














            Required, but never shown












            Required, but never shown







            Required, but never shown

































            Required, but never shown














            Required, but never shown












            Required, but never shown







            Required, but never shown







            Popular posts from this blog

            Can a sorcerer learn a 5th-level spell early by creating spell slots using the Font of Magic feature?

            Does disintegrating a polymorphed enemy still kill it after the 2018 errata?

            A Topological Invariant for $pi_3(U(n))$